matheraum.de
Raum für Mathematik
Offene Informations- und Nachhilfegemeinschaft

Für Schüler, Studenten, Lehrer, Mathematik-Interessierte.
Hallo Gast!einloggen | registrieren ]
Startseite · Forum · Wissen · Kurse · Mitglieder · Team · Impressum
Forenbaum
^ Forenbaum
Status Mathe
  Status Schulmathe
    Status Primarstufe
    Status Mathe Klassen 5-7
    Status Mathe Klassen 8-10
    Status Oberstufenmathe
    Status Mathe-Wettbewerbe
    Status Sonstiges
  Status Hochschulmathe
    Status Uni-Analysis
    Status Uni-Lin. Algebra
    Status Algebra+Zahlentheo.
    Status Diskrete Mathematik
    Status Fachdidaktik
    Status Finanz+Versicherung
    Status Logik+Mengenlehre
    Status Numerik
    Status Uni-Stochastik
    Status Topologie+Geometrie
    Status Uni-Sonstiges
  Status Mathe-Vorkurse
    Status Organisatorisches
    Status Schule
    Status Universität
  Status Mathe-Software
    Status Derive
    Status DynaGeo
    Status FunkyPlot
    Status GeoGebra
    Status LaTeX
    Status Maple
    Status MathCad
    Status Mathematica
    Status Matlab
    Status Maxima
    Status MuPad
    Status Taschenrechner

Gezeigt werden alle Foren bis zur Tiefe 2

Navigation
 Startseite...
 Neuerdings beta neu
 Forum...
 vorwissen...
 vorkurse...
 Werkzeuge...
 Nachhilfevermittlung beta...
 Online-Spiele beta
 Suchen
 Verein...
 Impressum
Das Projekt
Server und Internetanbindung werden durch Spenden finanziert.
Organisiert wird das Projekt von unserem Koordinatorenteam.
Hunderte Mitglieder helfen ehrenamtlich in unseren moderierten Foren.
Anbieter der Seite ist der gemeinnützige Verein "Vorhilfe.de e.V.".
Partnerseiten
Dt. Schulen im Ausland: Mathe-Seiten:Weitere Fächer:

Open Source FunktionenplotterFunkyPlot: Kostenloser und quelloffener Funktionenplotter für Linux und andere Betriebssysteme
StartseiteMatheForenElektrotechnikKegelwiderstand berechnen
Foren für weitere Schulfächer findest Du auf www.vorhilfe.de z.B. Philosophie • Religion • Kunst • Musik • Sport • Pädagogik
Forum "Elektrotechnik" - Kegelwiderstand berechnen
Kegelwiderstand berechnen < Elektrotechnik < Ingenieurwiss. < Vorhilfe
Ansicht: [ geschachtelt ] | ^ Forum "Elektrotechnik"  | ^^ Alle Foren  | ^ Forenbaum  | Materialien

Kegelwiderstand berechnen: Rückfrage
Status: (Frage) beantwortet Status 
Datum: 14:24 Di 23.02.2016
Autor: Ulquiorra

Aufgabe
In der Anordnung einer elektrischen Anlage befindet sich ein leitfähiges Übergangsstück mit den gegebenen Werten.

a) Berechnen Sie den Gesamtwiderstand der Anordnung.
b) Zeichnen Sie den qualitativen Verlauf der Stromdichte in x-Richtung.
c) Ermitteln Sie den Verlauf des Feldstärkebetrags entlang der Mittellinie.

[Dateianhang nicht öffentlich]


Hallo,
ich hab einen Teil der Skizze ausgelassen, da der für mich nicht wichtig war und einfach zu berechnen und mal nur den wesentlichen Teil gezeichnet.
a) Also für die beiden anderen Widerstände hat mein Dozent diese Formel für die Berechnung benutzt. Die beiden anderen Widerstände waren einfach nur Zylinder und somit ziemlich einfach zu berechnen mit der Formel.
R= [mm] \bruch{l}{\kappa * A} [/mm]
Der Radius dieses Widerstandes war jedoch abhängig von der Länge, also von x. (In der Aufgabe ist KEIN Koordinatensystem und deswegen habe ich es auch weggelassen)

Ich hätte nun eine Funktion für r(x) aufgestellt um sie daraufhin für r in A = [mm] \pi [/mm] * r² einzusetzen und dann A in der oberen Gleichung ersetzen.

Nun mein Dozent hat das mit dem Integral gemacht und ich kann gerade nicht nachvollziehen warum.

Weg des Dozenten:
R = [mm] \integral{\bruch{1}{\kappa * \pi} \bruch{1}{r(x)^{2}} dx} [/mm]

Und dann erst hat er r(x) mit der Funktion, die wir vorher bestimmt hatten ersetzt. Und mit anschließender Substitution integriert.

Ich hab mal den Rechenweg weggelassen, weil ich es mathematisch nachvollziehen kann, aber nicht methodisch. Ich versteh also nicht, was ihn daran hinderte schon in
R= [mm] \bruch{l}{\kappa * \pi * r(x)^{2}} [/mm]
das r(x) zu ersetzen

b)
Die Zeichnung lass ich mal weg, aber grob kann man sagen: Je größer der Radius (Fläche), desto geringer die Stromdichte. Aufgrund von J = [mm] \bruch{I}{A} [/mm]

c)
Bei c würde ich behaupten es wäre wie b, aufgrund dem [mm] \bruch{1}{r²} [/mm] in der Formel für E.

Edit: Was mir aufgefallen ist. Wenn ich nach einer Klammer [Alt Gr] + [2] verwende, wird das Quadrat nicht angezeigt, aber wenn ich das nach einem Buchstaben benutze schon.

Mit freundlichen Grüßen Ulq

Dateianhänge:
Anhang Nr. 1 (Typ: png) [nicht öffentlich]
        
Bezug
Kegelwiderstand berechnen: Antwort
Status: (Antwort) fertig Status 
Datum: 15:11 Di 23.02.2016
Autor: leduart

Hallo
R ändert sich dich mit x? in Gedanken kannst du den Kegel in lauter hintereinander geschaltete Zylinder der Höhe dx zerlegen und alle diese Widerstände addieren. Genau das tut das Integral  Wenn du r(x) einsetzt in R bekommst du doch für [mm] x=x_0 [/mm]  einfach den Widerstand eines  Zylinders der Höhe [mm] x_0 [/mm]
Gruß leduart

Bezug
                
Bezug
Kegelwiderstand berechnen: Mitteilung
Status: (Mitteilung) Reaktion unnötig Status 
Datum: 15:26 Di 23.02.2016
Autor: GvC

Es sollte nicht unerwähnt bleiben, dass die geschilderte Berechnungsweise nur ein näherungsweise richtiges Ergebnis liefert. Der Grund ist, dass die Querschnittsfläche jedes infinitesimal kurzen Elementes nicht senkrecht vom Strom durchflossen wird. Die Aufgabenteile b) und c) weisen darauf hin, dass im Aufgabenteil a) gewisse Vereinfachungen getroffen wurden.

Bezug
        
Bezug
Kegelwiderstand berechnen: Antwort
Status: (Antwort) fertig Status 
Datum: 22:49 Di 23.02.2016
Autor: HJKweseleit


> In der Anordnung einer elektrischen Anlage befindet sich
> ein leitfähiges Übergangsstück mit den gegebenen
> Werten.
>  
> a) Berechnen Sie den Gesamtwiderstand der Anordnung.
>  b) Zeichnen Sie den qualitativen Verlauf der Stromdichte
> in x-Richtung.
>  c) Ermitteln Sie den Verlauf des Feldstärkebetrags
> entlang der Mittellinie.
>  
> [Dateianhang nicht öffentlich]
>  
> Hallo,
>  ich hab einen Teil der Skizze ausgelassen, da der für
> mich nicht wichtig war und einfach zu berechnen und mal nur
> den wesentlichen Teil gezeichnet.
>  a) Also für die beiden anderen Widerstände hat mein
> Dozent diese Formel für die Berechnung benutzt. Die beiden
> anderen Widerstände waren einfach nur Zylinder und somit
> ziemlich einfach zu berechnen mit der Formel.
>  R= [mm]\bruch{l}{\kappa * A}[/mm]
> Der Radius dieses Widerstandes war jedoch abhängig von der
> Länge, also von x. (In der Aufgabe ist KEIN
> Koordinatensystem und deswegen habe ich es auch
> weggelassen)
>  
> Ich hätte nun eine Funktion für r(x) aufgestellt um sie
> daraufhin für r in A = [mm]\pi[/mm] * r² einzusetzen und dann A in
> der oberen Gleichung ersetzen.
>  
> Nun mein Dozent hat das mit dem Integral gemacht und ich
> kann gerade nicht nachvollziehen warum.
>  
> Weg des Dozenten:
>  R = [mm]\integral{\bruch{1}{\kappa * \pi} \bruch{1}{r(x)^{2}} dx}[/mm]
>  
> Und dann erst hat er r(x) mit der Funktion, die wir vorher
> bestimmt hatten ersetzt. Und mit anschließender
> Substitution integriert.
>  
> Ich hab mal den Rechenweg weggelassen, weil ich es
> mathematisch nachvollziehen kann, aber nicht methodisch.
> Ich versteh also nicht, was ihn daran hinderte schon in
> R= [mm]\bruch{l}{\kappa * \pi * r(x)^{2}}[/mm]
> das r(x) zu ersetzen




Natürlich könnte er das r(x) ersetzen, und das tut er ja auch im Integral durch den von dir erwähnten Term.

Aber welches r(x) könnte er ohne Integral denn nehmen? Zu Anfang ist [mm] r=0,5*d_1, [/mm] am Ende ist [mm] r=0,5*d_2, [/mm] zwischendurch sind es andere Werte. Wenn R proportional zu r wäre, könnte er den Mittelwert von beiden nehmen - das ist aber hier nicht der Fall.

(Ein Beispiel dazu: Wenn du 5 km weit mit 15 km/h fährst und dann 5 km weit mit 30 km/h, brauchst du für die 10 km 20 min + 10 min = 30 min, bist somit durchschnittlich mit 20 km/h gefahren und nicht mit dem Mittelwert 22,5 km/h.)

Auch das einsetzen der von  dir erwähnten Formel führt nicht zum Ziel, denn sie enthält das x, und x ist zu Anfang 0, am Ende l, und für beide dieser Werte wäre R falsch berechnet. Das Integral sucht dir hier sozusagen den passenden "Mittelwert" heraus.

Für jedes x wird der Radius ermittelt, daraus die Fläche A berechnet und der Widerstand für einen Zylinder mit dieser Fläche und der Dicke l=dx errechnet. Weil diese Widerstände vom Strom nacheinander durchlaufen werden, hast du eine Reihenschaltung, und deshalb summiertst du diese Widerstände alle auf, indem du ein großes S [mm] (\integral) [/mm] davor schreibst. Nichts anderes ist das Integrieren: eine Summe aus gaaaanz vielen gaaaanz kleinen Summanden, von denen der Grenzwert ermittelt wird.


[mm] \fbox{Solche Integrale solltest du unbedingt verstehen, die halbe Experimentalphysik besteht aus der Zusammenstellung und Berechnung solcher Integrale!} [/mm]




>  
> b)
>  Die Zeichnung lass ich mal weg, aber grob kann man sagen:
> Je größer der Radius (Fläche), desto geringer die
> Stromdichte. Aufgrund von J = [mm]\bruch{I}{A}[/mm]
>  



Richtig. Und für A hast du ja die besagte Formel mit x.


> c)
>  Bei c würde ich behaupten es wäre wie b, aufgrund dem
> [mm]\bruch{1}{r²}[/mm] in der Formel für E.



Gut beobachtet!
Wegen [mm] U=R*I=\bruch{l}{\kappa*A}*I [/mm] gilt für die kleine Dicke l=dx: [mm] U=R*I=\bruch{dx}{\kappa*A}*I [/mm]
Die Feldstärke ist aber gerade E=U/d, wobei hier d=dx ist, und damit wird [mm] E=\bruch{I}{\kappa*A}, [/mm] wobei du für A wieder die Formel mit x einsetzen solltest.
  


>  
> Edit: Was mir aufgefallen ist. Wenn ich nach einer Klammer
> [Alt Gr] + [2] verwende, wird das Quadrat nicht angezeigt,
> aber wenn ich das nach einem Buchstaben benutze schon.
>  
> Mit freundlichen Grüßen Ulq



Du kannst hochstellen mit dem Dach-Zeichen links oben auf der Tastatur, auf dem auch das Grad-Zeichen ° zu finden ist. wenn du mehr als ein Zeichen hochstellen willst, musst du alles in Mengenklammern (AltGr+7 bzw 0) setzen. Halte den Cursor mal kurz auf: [mm] e^3 [/mm] und: [mm] wert^{Exponent}. [/mm] (die Dollar-Zeichen kannst du weglassen)

Tiefstellen geht genau so, indem du das Dach durch SHIFT+Minuszeichen (also den Unterstrich) austauscht. Manchmal gibts aber Ärger, wenn das Programm nicht weiß, was zusammen gehört. Geh mal mit dem Cursor auf das H von [mm] H_2 [/mm] O, auf H_2O (da tut sich nichts) und auf [mm] H_{2}O. [/mm]


Bezug
                
Bezug
Kegelwiderstand berechnen: Mitteilung
Status: (Mitteilung) Reaktion unnötig Status 
Datum: 01:25 Mi 24.02.2016
Autor: Ulquiorra

Ok danke an alle; habs jetzt verstanden. Aber wie krieg ich den Status auf beantwortet? Oder habe ich irgendwo eine Frage eurerseits übersehen?

Ulq

Bezug
Ansicht: [ geschachtelt ] | ^ Forum "Elektrotechnik"  | ^^ Alle Foren  | ^ Forenbaum  | Materialien


^ Seitenanfang ^
www.matheraum.de
[ Startseite | Forum | Wissen | Kurse | Mitglieder | Team | Impressum ]